Computability Theory Question.

W = { <M> | M is a DFA and L(M) is NOT a finite set}. Prove that W is a decidable language. Please write the algorithm for the machine that will decide this language. Also please add a comment to each step of the algorithm.

Join Matchmaticians Affiliate Marketing Program to earn up to 50% commission on every question your affiliated users ask or answer.
  • closed
  • 381 views
  • $2.00

Related Questions